LSAT and Law School Admissions Forum

Get expert LSAT preparation and law school admissions advice from PowerScore Test Preparation.

User avatar
 Dave Killoran
PowerScore Staff
  • PowerScore Staff
  • Posts: 5853
  • Joined: Mar 25, 2011
|
#85539
Complete Question Explanation
(The complete setup for this game can be found here: lsat/viewtopic.php?f=307&t=9348)

The correct answer choice is (B)

If G is inspected on Tuesday, then G must be inspected on Tuesday morning and J must be inspected on Tuesday afternoon.

G3-Q16-d1.png

Accordingly, answer choices (C) and (D) can immediately be eliminated because neither reflects the fact that J must be inspected on Tuesday afternoon.

Answer choice (A) can be eliminated because, from the contrapositive of the fourth rule, if L is inspected in the afternoon, then Z must also be inspected in the afternoon.

Answer choice (E) can be eliminated for two reasons: first, from the first rule L cannot be inspected on Wednesday, and, second, from the contrapositive of the fourth rule, if L is inspected in the afternoon, then Z must also be inspected in the afternoon.

Thus, answer choice (B) is the correct answer.
You do not have the required permissions to view the files attached to this post.

Get the most out of your LSAT Prep Plus subscription.

Analyze and track your performance with our Testing and Analytics Package.